Difference between revisions of "2009 AIME II Problems/Problem 15"
Michelangelo (talk | contribs) (→See Also) |
5849206328x (talk | contribs) |
||
Line 2: | Line 2: | ||
Let <math>\overline{MN}</math> be a diameter of a circle with diameter 1. Let <math>A</math> and <math>B</math> be points on one of the semicircular arcs determined by <math>\overline{MN}</math> such that <math>A</math> is the midpoint of the semicircle and <math>MB=\frac{3}5</math>. Point <math>C</math> lies on the other semicircular arc. Let <math>d</math> be the length of the line segment whose endpoints are the intersections of diameter <math>\overline{MN}</math> with chords <math>\overline{AC}</math> and <math>\overline{BC}</math>. The largest possible value of <math>d</math> can be written in the form <math> r-s\sqrt{t} </math>, where <math>r, s</math> and <math>t</math> are positive integers and <math>t</math> is not divisible by the square of any prime. Find <math>r+s+t</math>. | Let <math>\overline{MN}</math> be a diameter of a circle with diameter 1. Let <math>A</math> and <math>B</math> be points on one of the semicircular arcs determined by <math>\overline{MN}</math> such that <math>A</math> is the midpoint of the semicircle and <math>MB=\frac{3}5</math>. Point <math>C</math> lies on the other semicircular arc. Let <math>d</math> be the length of the line segment whose endpoints are the intersections of diameter <math>\overline{MN}</math> with chords <math>\overline{AC}</math> and <math>\overline{BC}</math>. The largest possible value of <math>d</math> can be written in the form <math> r-s\sqrt{t} </math>, where <math>r, s</math> and <math>t</math> are positive integers and <math>t</math> is not divisible by the square of any prime. Find <math>r+s+t</math>. | ||
− | == | + | == Solutions == |
+ | ===Solution 1=== | ||
Let <math>O</math> be the center of the circle. Define <math>\angle{MOC}=t</math>, <math>\angle{BOA}=2a</math>, and let <math>BC</math> and <math>AC</math> intersect <math>MN</math> at points <math>X</math> and <math>Y</math>, respectively. We will express the length of <math>XY</math> as a function of <math>t</math> and maximize that function in the interval <math>[0, \pi]</math>. | Let <math>O</math> be the center of the circle. Define <math>\angle{MOC}=t</math>, <math>\angle{BOA}=2a</math>, and let <math>BC</math> and <math>AC</math> intersect <math>MN</math> at points <math>X</math> and <math>Y</math>, respectively. We will express the length of <math>XY</math> as a function of <math>t</math> and maximize that function in the interval <math>[0, \pi]</math>. | ||
Line 84: | Line 85: | ||
so the maximum length of <math>XY</math> is <math>\sin(a)\times\frac{\sin(t')}{\sin(t' + a) + \cos(a)} = 7 - 4\sqrt{3}</math>, and the answer is <math>7 + 4 + 3 = \boxed{014}</math>. | so the maximum length of <math>XY</math> is <math>\sin(a)\times\frac{\sin(t')}{\sin(t' + a) + \cos(a)} = 7 - 4\sqrt{3}</math>, and the answer is <math>7 + 4 + 3 = \boxed{014}</math>. | ||
+ | |||
+ | ===Solution 2=== | ||
+ | <asy> | ||
+ | unitsize(144); | ||
+ | pair A, B, C, M, n; | ||
+ | A = (0,1); B = (-7/25, 24/25); C=(1/7,-4*sqrt(3)/7); M = (-1,0); n = (1,0); | ||
+ | pair [] D = intersectionpoints(A--C,M--n); pair [] e = intersectionpoints(B--C,M--n); | ||
+ | |||
+ | draw(circle((0,0),1)); | ||
+ | draw(M--n--B--M--A--n--C--A--B--C--cycle); | ||
+ | |||
+ | label("$A$",A,N); label("$B$",B,NNW); label("$M$",M,W); label("$C$",C,SSE); label("$N$",n,E); | ||
+ | label("$D$",D[0],SE); label("$E$",e[0],SW); | ||
+ | label("$x$",(M+C)/2,SW); label("$y$",(n+C)/2,SE); | ||
+ | </asy> | ||
+ | |||
+ | Suppose <math>\overline{AC}</math> and <math>\overline{BC}</math> intersect <math>\overline{MN}</math> at <math>D</math> and <math>E</math>, respectively, and let <math>MC = x</math> and <math>NC = y</math>. Since <math>A</math> is the midpoint of arc <math>MN</math>, <math>\overline{CA}</math> bisects <math>\angle MCN</math>, and we get | ||
+ | <cmath>\frac{MC}{MD} = \frac{NC}{ND}\Rightarrow MD = \frac{x}{x + y}.</cmath> | ||
+ | To find <math>ME</math>, we note that <math>\triangle BND\sim\triangle MCD</math> and <math>\triangle BMD\sim\triangle NCD</math>, so | ||
+ | <cmath>\begin{align*} | ||
+ | \frac{BN}{NE} &= \frac{MC}{CE} \\ | ||
+ | \frac{ME}{BM} &= \frac{CE}{NC}. | ||
+ | \end{align*}</cmath> | ||
+ | Writing <math>NE = 1 - ME</math>, we can substitute known values and multiply the equations to get | ||
+ | <cmath>\frac{4(ME)}{3 - 3(ME)} = \frac{x}{y}\Rightarrow ME = \frac{3x}{3x + 4y}.</cmath> | ||
+ | The value we wish to maximize is | ||
+ | <cmath>\begin{align*} | ||
+ | DE &= MD - ME \\ | ||
+ | &= \frac{x}{x + y} - \frac{3x}{3x + 4y} \\ | ||
+ | &= \frac{xy}{3x^2 + 7xy + 4y^2} \\ | ||
+ | &= \frac{1}{3(x/y) + 4(y/x) + 7}. | ||
+ | \end{align*}</cmath> | ||
+ | By the AM-GM inequality, <math>3(x/y) + 4(y/x)\geq 2\sqrt{12} = 4\sqrt{3}</math>, so | ||
+ | <cmath>DE\leq \frac{1}{4\sqrt{3} + 7} = 7 - 4\sqrt{3},</cmath> | ||
+ | giving the answer of <math>7 + 4 + 3 = \boxed{014}</math>. Equality is achieved when <math>3(x/y) = 4(y/x)</math> subject to the conditions <math>x^2 + y^2 = 1</math> and <math>x > 0 > y</math>, which occurs for <math>(x,y) = \left(\frac{2\sqrt{7}}{7}, -\frac{\sqrt{21}}{7}\right)</math>. | ||
==See Also== | ==See Also== | ||
{{AIME box|year=2009|n=II|num-b=14|after=Last Problem}} | {{AIME box|year=2009|n=II|num-b=14|after=Last Problem}} | ||
{{MAA Notice}} | {{MAA Notice}} |
Revision as of 11:04, 4 April 2014
Problem
Let be a diameter of a circle with diameter 1. Let and be points on one of the semicircular arcs determined by such that is the midpoint of the semicircle and . Point lies on the other semicircular arc. Let be the length of the line segment whose endpoints are the intersections of diameter with chords and . The largest possible value of can be written in the form , where and are positive integers and is not divisible by the square of any prime. Find .
Solutions
Solution 1
Let be the center of the circle. Define , , and let and intersect at points and , respectively. We will express the length of as a function of and maximize that function in the interval .
Let be the foot of the perpendicular from to . We compute as follows.
(a) By the Extended Law of Sines in triangle , we have
(b) Note that and . Since and are similar right triangles, we have , and hence,
(c) We have and , and hence by the Law of Sines,
(d) Multiplying (a), (b), and (c), we have
,
which is a function of (and the constant ). Differentiating this with respect to yields
,
and the numerator of this is
,
which vanishes when . Therefore, the length of is maximized when , where is the value in that satisfies .
Note that
,
so . We compute
,
so the maximum length of is , and the answer is .
Solution 2
Suppose and intersect at and , respectively, and let and . Since is the midpoint of arc , bisects , and we get To find , we note that and , so Writing , we can substitute known values and multiply the equations to get The value we wish to maximize is By the AM-GM inequality, , so giving the answer of . Equality is achieved when subject to the conditions and , which occurs for .
See Also
2009 AIME II (Problems • Answer Key • Resources) | ||
Preceded by Problem 14 |
Followed by Last Problem | |
1 • 2 • 3 • 4 • 5 • 6 • 7 • 8 • 9 • 10 • 11 • 12 • 13 • 14 • 15 | ||
All AIME Problems and Solutions |
The problems on this page are copyrighted by the Mathematical Association of America's American Mathematics Competitions.